The $n$-th root of $2$ is irrational for $n>1$, and $lim_{ntoinfty}sqrt[n]{2}=1$. Is the latter fact a...












-1














So I was looking at a theorem that popped up into my head:




The $n$-th root of $2$ always being irrational if $n$ is greater than $1$.




I also noticed that




The limit of the $n$-th root of $2$, as $n$ approaches infinity, is $1$.




Is there a connection between this "$n$-th root of $2$ Theorem" and the limit I mentioned?




Can I consider that limit a corollary to the theorem?











share|cite|improve this question




















  • 3




    Not really, no. Irrationality doesn't tend to interact well with limits; perhaps there' something more specific you have in mind.
    – T. Bongers
    Nov 28 at 23:35










  • If $n$ is irrational then the nth root of $2$ can be rational.
    – kingW3
    Nov 28 at 23:40












  • Just so I know, how do I keep the revisions? It sounds stupid, but it wouldn't let me keep the revisions that were made.
    – Xavier Stanton
    Nov 29 at 0:18










  • Also, how do you make the formulas?
    – Xavier Stanton
    Nov 29 at 0:19










  • We use MathJax to typeset formulas. A tutorial is here. To see the edit history you can click on the "edited xxx ago" in the center bottom of the post.
    – Ross Millikan
    Nov 29 at 0:39
















-1














So I was looking at a theorem that popped up into my head:




The $n$-th root of $2$ always being irrational if $n$ is greater than $1$.




I also noticed that




The limit of the $n$-th root of $2$, as $n$ approaches infinity, is $1$.




Is there a connection between this "$n$-th root of $2$ Theorem" and the limit I mentioned?




Can I consider that limit a corollary to the theorem?











share|cite|improve this question




















  • 3




    Not really, no. Irrationality doesn't tend to interact well with limits; perhaps there' something more specific you have in mind.
    – T. Bongers
    Nov 28 at 23:35










  • If $n$ is irrational then the nth root of $2$ can be rational.
    – kingW3
    Nov 28 at 23:40












  • Just so I know, how do I keep the revisions? It sounds stupid, but it wouldn't let me keep the revisions that were made.
    – Xavier Stanton
    Nov 29 at 0:18










  • Also, how do you make the formulas?
    – Xavier Stanton
    Nov 29 at 0:19










  • We use MathJax to typeset formulas. A tutorial is here. To see the edit history you can click on the "edited xxx ago" in the center bottom of the post.
    – Ross Millikan
    Nov 29 at 0:39














-1












-1








-1







So I was looking at a theorem that popped up into my head:




The $n$-th root of $2$ always being irrational if $n$ is greater than $1$.




I also noticed that




The limit of the $n$-th root of $2$, as $n$ approaches infinity, is $1$.




Is there a connection between this "$n$-th root of $2$ Theorem" and the limit I mentioned?




Can I consider that limit a corollary to the theorem?











share|cite|improve this question















So I was looking at a theorem that popped up into my head:




The $n$-th root of $2$ always being irrational if $n$ is greater than $1$.




I also noticed that




The limit of the $n$-th root of $2$, as $n$ approaches infinity, is $1$.




Is there a connection between this "$n$-th root of $2$ Theorem" and the limit I mentioned?




Can I consider that limit a corollary to the theorem?








limits elementary-number-theory






share|cite|improve this question















share|cite|improve this question













share|cite|improve this question




share|cite|improve this question








edited Nov 29 at 4:55









Jyrki Lahtonen

108k12166366




108k12166366










asked Nov 28 at 23:33









Xavier Stanton

330211




330211








  • 3




    Not really, no. Irrationality doesn't tend to interact well with limits; perhaps there' something more specific you have in mind.
    – T. Bongers
    Nov 28 at 23:35










  • If $n$ is irrational then the nth root of $2$ can be rational.
    – kingW3
    Nov 28 at 23:40












  • Just so I know, how do I keep the revisions? It sounds stupid, but it wouldn't let me keep the revisions that were made.
    – Xavier Stanton
    Nov 29 at 0:18










  • Also, how do you make the formulas?
    – Xavier Stanton
    Nov 29 at 0:19










  • We use MathJax to typeset formulas. A tutorial is here. To see the edit history you can click on the "edited xxx ago" in the center bottom of the post.
    – Ross Millikan
    Nov 29 at 0:39














  • 3




    Not really, no. Irrationality doesn't tend to interact well with limits; perhaps there' something more specific you have in mind.
    – T. Bongers
    Nov 28 at 23:35










  • If $n$ is irrational then the nth root of $2$ can be rational.
    – kingW3
    Nov 28 at 23:40












  • Just so I know, how do I keep the revisions? It sounds stupid, but it wouldn't let me keep the revisions that were made.
    – Xavier Stanton
    Nov 29 at 0:18










  • Also, how do you make the formulas?
    – Xavier Stanton
    Nov 29 at 0:19










  • We use MathJax to typeset formulas. A tutorial is here. To see the edit history you can click on the "edited xxx ago" in the center bottom of the post.
    – Ross Millikan
    Nov 29 at 0:39








3




3




Not really, no. Irrationality doesn't tend to interact well with limits; perhaps there' something more specific you have in mind.
– T. Bongers
Nov 28 at 23:35




Not really, no. Irrationality doesn't tend to interact well with limits; perhaps there' something more specific you have in mind.
– T. Bongers
Nov 28 at 23:35












If $n$ is irrational then the nth root of $2$ can be rational.
– kingW3
Nov 28 at 23:40






If $n$ is irrational then the nth root of $2$ can be rational.
– kingW3
Nov 28 at 23:40














Just so I know, how do I keep the revisions? It sounds stupid, but it wouldn't let me keep the revisions that were made.
– Xavier Stanton
Nov 29 at 0:18




Just so I know, how do I keep the revisions? It sounds stupid, but it wouldn't let me keep the revisions that were made.
– Xavier Stanton
Nov 29 at 0:18












Also, how do you make the formulas?
– Xavier Stanton
Nov 29 at 0:19




Also, how do you make the formulas?
– Xavier Stanton
Nov 29 at 0:19












We use MathJax to typeset formulas. A tutorial is here. To see the edit history you can click on the "edited xxx ago" in the center bottom of the post.
– Ross Millikan
Nov 29 at 0:39




We use MathJax to typeset formulas. A tutorial is here. To see the edit history you can click on the "edited xxx ago" in the center bottom of the post.
– Ross Millikan
Nov 29 at 0:39










3 Answers
3






active

oldest

votes


















1














As far as I understand your question, no. The sequence



$$2sqrt{2},2root3of2,2root4of2,cdots$$



(multiplying every term of your nth-root-of-$2$ sequence by $2$) contains only irrational terms and tends to $2$, while the sequence



$$1+1,1+frac{1}{2},1+frac{1}{3},cdots$$



contains only rational terms and tends to $1$. The two properties (containing irrational terms and tending to 1) have nothing to do with one another.






share|cite|improve this answer





























    0














    The two things are not related, for the limit indeed we have that



    $$sqrt[n] 2=e^{frac{log 2}n}to 1$$



    since $frac{log 2}nto 0$, and the result is not related to any hypotesis about the irrationality of $sqrt[n] 2$.






    share|cite|improve this answer





























      0














      The $n^{th}$ root of any positive number tends toward $1$ as $n to infty$. This is regardless of whether the roots are rational or irrational. For any given positive number there will only be a finite number of rational roots, but it can be arbitrarily large. Find some $k$ with many factors, like one of the highly composite numbers. Now form $1.01^k$. That will have as many rational roots as $k$ has factors.






      share|cite|improve this answer





















      • Thanks, that's interesting.
        – Xavier Stanton
        Nov 28 at 23:47











      Your Answer





      StackExchange.ifUsing("editor", function () {
      return StackExchange.using("mathjaxEditing", function () {
      StackExchange.MarkdownEditor.creationCallbacks.add(function (editor, postfix) {
      StackExchange.mathjaxEditing.prepareWmdForMathJax(editor, postfix, [["$", "$"], ["\\(","\\)"]]);
      });
      });
      }, "mathjax-editing");

      StackExchange.ready(function() {
      var channelOptions = {
      tags: "".split(" "),
      id: "69"
      };
      initTagRenderer("".split(" "), "".split(" "), channelOptions);

      StackExchange.using("externalEditor", function() {
      // Have to fire editor after snippets, if snippets enabled
      if (StackExchange.settings.snippets.snippetsEnabled) {
      StackExchange.using("snippets", function() {
      createEditor();
      });
      }
      else {
      createEditor();
      }
      });

      function createEditor() {
      StackExchange.prepareEditor({
      heartbeatType: 'answer',
      autoActivateHeartbeat: false,
      convertImagesToLinks: true,
      noModals: true,
      showLowRepImageUploadWarning: true,
      reputationToPostImages: 10,
      bindNavPrevention: true,
      postfix: "",
      imageUploader: {
      brandingHtml: "Powered by u003ca class="icon-imgur-white" href="https://imgur.com/"u003eu003c/au003e",
      contentPolicyHtml: "User contributions licensed under u003ca href="https://creativecommons.org/licenses/by-sa/3.0/"u003ecc by-sa 3.0 with attribution requiredu003c/au003e u003ca href="https://stackoverflow.com/legal/content-policy"u003e(content policy)u003c/au003e",
      allowUrls: true
      },
      noCode: true, onDemand: true,
      discardSelector: ".discard-answer"
      ,immediatelyShowMarkdownHelp:true
      });


      }
      });














      draft saved

      draft discarded


















      StackExchange.ready(
      function () {
      StackExchange.openid.initPostLogin('.new-post-login', 'https%3a%2f%2fmath.stackexchange.com%2fquestions%2f3017910%2fthe-n-th-root-of-2-is-irrational-for-n1-and-lim-n-to-infty-sqrtn2%23new-answer', 'question_page');
      }
      );

      Post as a guest















      Required, but never shown

























      3 Answers
      3






      active

      oldest

      votes








      3 Answers
      3






      active

      oldest

      votes









      active

      oldest

      votes






      active

      oldest

      votes









      1














      As far as I understand your question, no. The sequence



      $$2sqrt{2},2root3of2,2root4of2,cdots$$



      (multiplying every term of your nth-root-of-$2$ sequence by $2$) contains only irrational terms and tends to $2$, while the sequence



      $$1+1,1+frac{1}{2},1+frac{1}{3},cdots$$



      contains only rational terms and tends to $1$. The two properties (containing irrational terms and tending to 1) have nothing to do with one another.






      share|cite|improve this answer


























        1














        As far as I understand your question, no. The sequence



        $$2sqrt{2},2root3of2,2root4of2,cdots$$



        (multiplying every term of your nth-root-of-$2$ sequence by $2$) contains only irrational terms and tends to $2$, while the sequence



        $$1+1,1+frac{1}{2},1+frac{1}{3},cdots$$



        contains only rational terms and tends to $1$. The two properties (containing irrational terms and tending to 1) have nothing to do with one another.






        share|cite|improve this answer
























          1












          1








          1






          As far as I understand your question, no. The sequence



          $$2sqrt{2},2root3of2,2root4of2,cdots$$



          (multiplying every term of your nth-root-of-$2$ sequence by $2$) contains only irrational terms and tends to $2$, while the sequence



          $$1+1,1+frac{1}{2},1+frac{1}{3},cdots$$



          contains only rational terms and tends to $1$. The two properties (containing irrational terms and tending to 1) have nothing to do with one another.






          share|cite|improve this answer












          As far as I understand your question, no. The sequence



          $$2sqrt{2},2root3of2,2root4of2,cdots$$



          (multiplying every term of your nth-root-of-$2$ sequence by $2$) contains only irrational terms and tends to $2$, while the sequence



          $$1+1,1+frac{1}{2},1+frac{1}{3},cdots$$



          contains only rational terms and tends to $1$. The two properties (containing irrational terms and tending to 1) have nothing to do with one another.







          share|cite|improve this answer












          share|cite|improve this answer



          share|cite|improve this answer










          answered Nov 28 at 23:37









          Carl Schildkraut

          11.1k11441




          11.1k11441























              0














              The two things are not related, for the limit indeed we have that



              $$sqrt[n] 2=e^{frac{log 2}n}to 1$$



              since $frac{log 2}nto 0$, and the result is not related to any hypotesis about the irrationality of $sqrt[n] 2$.






              share|cite|improve this answer


























                0














                The two things are not related, for the limit indeed we have that



                $$sqrt[n] 2=e^{frac{log 2}n}to 1$$



                since $frac{log 2}nto 0$, and the result is not related to any hypotesis about the irrationality of $sqrt[n] 2$.






                share|cite|improve this answer
























                  0












                  0








                  0






                  The two things are not related, for the limit indeed we have that



                  $$sqrt[n] 2=e^{frac{log 2}n}to 1$$



                  since $frac{log 2}nto 0$, and the result is not related to any hypotesis about the irrationality of $sqrt[n] 2$.






                  share|cite|improve this answer












                  The two things are not related, for the limit indeed we have that



                  $$sqrt[n] 2=e^{frac{log 2}n}to 1$$



                  since $frac{log 2}nto 0$, and the result is not related to any hypotesis about the irrationality of $sqrt[n] 2$.







                  share|cite|improve this answer












                  share|cite|improve this answer



                  share|cite|improve this answer










                  answered Nov 28 at 23:37









                  gimusi

                  1




                  1























                      0














                      The $n^{th}$ root of any positive number tends toward $1$ as $n to infty$. This is regardless of whether the roots are rational or irrational. For any given positive number there will only be a finite number of rational roots, but it can be arbitrarily large. Find some $k$ with many factors, like one of the highly composite numbers. Now form $1.01^k$. That will have as many rational roots as $k$ has factors.






                      share|cite|improve this answer





















                      • Thanks, that's interesting.
                        – Xavier Stanton
                        Nov 28 at 23:47
















                      0














                      The $n^{th}$ root of any positive number tends toward $1$ as $n to infty$. This is regardless of whether the roots are rational or irrational. For any given positive number there will only be a finite number of rational roots, but it can be arbitrarily large. Find some $k$ with many factors, like one of the highly composite numbers. Now form $1.01^k$. That will have as many rational roots as $k$ has factors.






                      share|cite|improve this answer





















                      • Thanks, that's interesting.
                        – Xavier Stanton
                        Nov 28 at 23:47














                      0












                      0








                      0






                      The $n^{th}$ root of any positive number tends toward $1$ as $n to infty$. This is regardless of whether the roots are rational or irrational. For any given positive number there will only be a finite number of rational roots, but it can be arbitrarily large. Find some $k$ with many factors, like one of the highly composite numbers. Now form $1.01^k$. That will have as many rational roots as $k$ has factors.






                      share|cite|improve this answer












                      The $n^{th}$ root of any positive number tends toward $1$ as $n to infty$. This is regardless of whether the roots are rational or irrational. For any given positive number there will only be a finite number of rational roots, but it can be arbitrarily large. Find some $k$ with many factors, like one of the highly composite numbers. Now form $1.01^k$. That will have as many rational roots as $k$ has factors.







                      share|cite|improve this answer












                      share|cite|improve this answer



                      share|cite|improve this answer










                      answered Nov 28 at 23:41









                      Ross Millikan

                      291k23196370




                      291k23196370












                      • Thanks, that's interesting.
                        – Xavier Stanton
                        Nov 28 at 23:47


















                      • Thanks, that's interesting.
                        – Xavier Stanton
                        Nov 28 at 23:47
















                      Thanks, that's interesting.
                      – Xavier Stanton
                      Nov 28 at 23:47




                      Thanks, that's interesting.
                      – Xavier Stanton
                      Nov 28 at 23:47


















                      draft saved

                      draft discarded




















































                      Thanks for contributing an answer to Mathematics Stack Exchange!


                      • Please be sure to answer the question. Provide details and share your research!

                      But avoid



                      • Asking for help, clarification, or responding to other answers.

                      • Making statements based on opinion; back them up with references or personal experience.


                      Use MathJax to format equations. MathJax reference.


                      To learn more, see our tips on writing great answers.





                      Some of your past answers have not been well-received, and you're in danger of being blocked from answering.


                      Please pay close attention to the following guidance:


                      • Please be sure to answer the question. Provide details and share your research!

                      But avoid



                      • Asking for help, clarification, or responding to other answers.

                      • Making statements based on opinion; back them up with references or personal experience.


                      To learn more, see our tips on writing great answers.




                      draft saved


                      draft discarded














                      StackExchange.ready(
                      function () {
                      StackExchange.openid.initPostLogin('.new-post-login', 'https%3a%2f%2fmath.stackexchange.com%2fquestions%2f3017910%2fthe-n-th-root-of-2-is-irrational-for-n1-and-lim-n-to-infty-sqrtn2%23new-answer', 'question_page');
                      }
                      );

                      Post as a guest















                      Required, but never shown





















































                      Required, but never shown














                      Required, but never shown












                      Required, but never shown







                      Required, but never shown

































                      Required, but never shown














                      Required, but never shown












                      Required, but never shown







                      Required, but never shown







                      Popular posts from this blog

                      To store a contact into the json file from server.js file using a class in NodeJS

                      Redirect URL with Chrome Remote Debugging Android Devices

                      Dieringhausen